Difference between revisions of "2019 AMC 12B Problems/Problem 2"

(Solution)
(Problem)
Line 1: Line 1:
 
==Problem==
 
==Problem==
 +
If n is not a prime than (n-2) must be a prime. Which of the following choices provide a counter example for this statement?
  
 
==Solution==
 
==Solution==

Revision as of 12:47, 14 February 2019

Problem

If n is not a prime than (n-2) must be a prime. Which of the following choices provide a counter example for this statement?

Solution

answer should be 27

See Also

2019 AMC 12B (ProblemsAnswer KeyResources)
Preceded by
Problem 1
Followed by
Problem 3
1 2 3 4 5 6 7 8 9 10 11 12 13 14 15 16 17 18 19 20 21 22 23 24 25
All AMC 12 Problems and Solutions

The problems on this page are copyrighted by the Mathematical Association of America's American Mathematics Competitions. AMC logo.png